Mock AIME 3 Pre 2005 Problems/Problem 13

Revision as of 07:53, 2 April 2007 by Solafidefarms (talk | contribs) (soln?)

Problem

$13.$ Let $S$ denote the value of the sum

$\left(\frac{2}{3}\right)^{2005} \cdot \sum_{k=1}^{2005} \frac{k^2}{2^k} \cdot {2005 \choose k}$

Determine the remainder obtained when $S$ is divided by $1000$.

Solution

{solution}